LSAT and Law School Admissions Forum

Get expert LSAT preparation and law school admissions advice from PowerScore Test Preparation.

 Administrator
PowerScore Staff
  • PowerScore Staff
  • Posts: 8917
  • Joined: Feb 02, 2011
|
#84799
Complete Question Explanation

The correct answer choice is (D).

Answer choice (A):

Answer choice (B):

Answer choice (C):

Answer choice (D): This is the correct answer choice.

Answer choice (E):

This explanation is still in progress. Please post any questions below!
User avatar
 jackielsat
  • Posts: 8
  • Joined: Jun 07, 2022
|
#95795
Could you please explain why B is wrong?
Is it because of the first part of the answer? ie the " initial judicial resistance"?
User avatar
 katehos
PowerScore Staff
  • PowerScore Staff
  • Posts: 184
  • Joined: Mar 31, 2022
|
#95805
Hi Jackielsat!

Great job - you've got it! Answer choice (B)'s mention of 'initial judicial resistance' is the reason why (B) is incorrect. The specific historians in question believe that 'this trend represented a gain for women, one that reflects changing views about democracy and property' (lines 6-8). So, we don't know anything about whether or not they acknowledge initial judicial resistance and can eliminate the answer (to be thorough, it's of course important to know if the historians' opinions are mentioned again; but, a further look at the passage reveals no extra information about what the historians' opinions on judicial resistance might be). Remember: in Must Be True questions, the entire answer choice must be true, so even part of the answer choice being false or unknown is enough reason to eliminate it.

Answer choice (D), on the other hand, pretty neatly summarizes the quote I mentioned, so we can fully support the answer with the text and know that it is correct!

I hope this helps :)
Kate

Get the most out of your LSAT Prep Plus subscription.

Analyze and track your performance with our Testing and Analytics Package.